Can I obtain the inverse Laplace transform using complex analysis?

Click For Summary
SUMMARY

The discussion centers on obtaining the inverse Laplace transform of the function \(\mathcal{L}^{-1}[\frac{e^{-5s}}{s^2-4}]\) using complex analysis. The initial calculation yields \(f(t)=\frac{1}{4}e^{2(t-5)}-\frac{1}{4}e^{-2(t-5)}\), which is incorrect. The correct result incorporates the Heaviside function, yielding \(f(t)=\theta(t-5)(\frac{1}{4}e^{2(t-5)}-\frac{1}{4}e^{-2(t-5)})\). The mistake arises from not applying the condition that \(\lim_{\operatorname{Re}(s) \to -\infty} F(s)e^{st} = 0\) for \(t < 5\).

PREREQUISITES
  • Complex analysis fundamentals
  • Understanding of Laplace transforms
  • Residue theorem application
  • Heaviside function properties
NEXT STEPS
  • Study the application of the residue theorem in complex analysis
  • Learn about the Heaviside function and its role in Laplace transforms
  • Explore the conditions for convergence in inverse Laplace transforms
  • Investigate advanced techniques in complex analysis for handling discontinuities
USEFUL FOR

Students and professionals in mathematics, particularly those studying differential equations and complex analysis, as well as anyone involved in signal processing or control theory.

LagrangeEuler
Messages
711
Reaction score
22
Homework Statement
1. Find inverse Laplace transform
[tex]\mathcal{L}^{-1}[\frac{e^{-5s}}{s^2-4}][/tex]
Relevant Equations
Inverse Laplace transform can be calculated as sum of residues of ##F(s)e^{st}##.
[tex]\mathcal{L}^{-1}[F(s)]=\sum^n_{k=1}Res[F(s)e^{st},s=\alpha_k][/tex]
\mathcal{L}^{-1}[\frac{e^{-5s}}{s^2-4}]=Res[e^{-5s}\frac{1}{s^2-4}e^{st},s=2]+Res[e^{-5s}\frac{1}{s^2-4}e^{st},s=-2]
From that I am getting
f(t)=\frac{1}{4}e^{2(t-5)}-\frac{1}{4}e^{-2(t-5)}. And this is not correct. Result should be
f(t)=\theta(t-5)(\frac{1}{4}e^{2(t-5)}-\frac{1}{4}e^{-2(t-5)})
where ##\theta## is Heaviside function. Where is the mistake?
 
Physics news on Phys.org
LagrangeEuler said:
Homework Statement:: 1. Find inverse Laplace transform
\mathcal{L}^{-1}[\frac{e^{-5s}}{s^2-4}]
Relevant Equations:: Inverse Laplace transform can be calculated as sum of residues of ##F(s)e^{st}##.

This is only valid if \lim_{\operatorname{Re}(s) \to -\infty} F(s)e^{st} = 0. That is not the case for t &lt; 5.
 
  • Like
Likes LagrangeEuler
Thank you. Is it a way to show this somehow? Or to use some version of complex analysis to get this?
 
So my mine question, in this case, is can I somehow obtain this result using complex analysis?
 
Question: A clock's minute hand has length 4 and its hour hand has length 3. What is the distance between the tips at the moment when it is increasing most rapidly?(Putnam Exam Question) Answer: Making assumption that both the hands moves at constant angular velocities, the answer is ## \sqrt{7} .## But don't you think this assumption is somewhat doubtful and wrong?

Similar threads

  • · Replies 10 ·
Replies
10
Views
2K
  • · Replies 1 ·
Replies
1
Views
1K
  • · Replies 7 ·
Replies
7
Views
2K
  • · Replies 4 ·
Replies
4
Views
3K
  • · Replies 3 ·
Replies
3
Views
3K
Replies
2
Views
2K
  • · Replies 1 ·
Replies
1
Views
1K
  • · Replies 1 ·
Replies
1
Views
967
  • · Replies 8 ·
Replies
8
Views
1K
Replies
1
Views
2K